Pharmacology View Exam PDF
Document Details
![FertileEnlightenment6050](https://quizgecko.com/images/avatars/avatar-13.webp)
Uploaded by FertileEnlightenment6050
Tags
Summary
This document contains a series of practice questions related to pharmacology, specifically focusing on antianginal drugs and antiarrhythmics. The questions cover various aspects of these medications, including their effects, side effects, and appropriate use in different clinical scenarios. This is a good resource for medical professionals.
Full Transcript
**Pharmacology View Exam (All Qs)** ***(View Exam: Pg 72)*** **Tests** **Antianginal drugs** **1.** What is the clinical term for angina caused by coronary vasospasm? A. Classic angina. B. Myocardial infarction. C. Prinzmetal angina. D. Unstable angina. **1. Correct answer = C.** **2.** Al...
**Pharmacology View Exam (All Qs)** ***(View Exam: Pg 72)*** **Tests** **Antianginal drugs** **1.** What is the clinical term for angina caused by coronary vasospasm? A. Classic angina. B. Myocardial infarction. C. Prinzmetal angina. D. Unstable angina. **1. Correct answer = C.** **2.** All of the following medications can be useful for managing stable angina in a patient with coronary artery disease except: A. Amlodipine. B. Atenolol. C. Immediate-release nifedipine. D. Isosorbide dinitrate. **2. Correct answer = C.** **3.** A 72-year-old male presents to the primary care clinic complaining of chest tightness and pressure that is increasing in severity and frequency. His current medications include atenolol, lisinopril, and nitroglycerin. Which intervention is most appropriate at this time? A. Add amlodipine. B. Initiate isosorbide mononitrate. C. Initiate ranolazine. **D. Refer the patient to the nearest emergency room for evaluation**. **3. Correct answer = D.** **4.** A 62-year-old patient with a history of asthma and vasospastic angina states that he gets chest pain both with exertion and at rest, about ten times per week. One sublingual nitroglycerin tablet always relieves his symptoms, but this medication gives him an awful headache every time he takes it. Which is the best option for improving his angina? A. Change to sublingual nitroglycerin spray. B. Add amlodipine. C. Add propranolol. D. Replace nitroglycerin with ranolazine. **4. Correct answer = B.** **5.** Which side **effect is associated with amlodipine**? A. Bradycardia. B. Cough. C. Edema. D. QT prolongation. **5. Correct answer = C.** **6.** A 58-year-old man presents to his primary care physician with a complaint of severe chest pain when he walks uphill to his home in cold weather. The pain disappears when he rests. After evaluation and discussion of treatment options, a decision is made to treat him with nitroglycerin. One year later, the patient returns complaining that his nitroglycerin works well when he takes it for an acute attack but that he is **now having more frequent attacks and would like something to prevent them**. Useful drugs for the prophylaxis of angina of effort include A. Amyl nitrite B. Diltiazem C. Esmolol D. Sublingual isosorbide dinitrate E\) Sublingual nitroglycerin **6. The answer is B.** **7.** A 65-year-old male experiences uncontrolled angina attacks that limit his ability to do **household chores. He is adherent to a maximized dose of β-blocker with a low heart rate and low blood pressure**. He was unable to tolerate an increase in isosorbide mononitrate due to headache. Which is the most appropriate addition to his antianginal therapy? A. Amlodipine. B. Aspirin. C. Ranolazine. D. Verapamil. **7. Correct answer = C.** **8.** A 68-year-old male with a history of angina had a MI last month, and an echocardiogram reveals heart failure with reduced ejection fraction. He was continued on his previous home medications (diltiazem, enalapril, and nitroglycerin), and atenolol was added at discharge. He has only had a few sporadic episodes of stable angina that are relieved with nitroglycerin or rest. What are eventual goals for optimizing this medication regimen? A. Add isosorbide mononitrate. B. Increase atenolol. C. Stop atenolol and increase diltiazem. D. Stop diltiazem and change atenolol to bisoprolol. **8. Correct answer = D** **9.** Which of the following medications would be safe to use in a patient taking ranolazine? A. Carbamazepine. B. Clarithromycin. C. Enalapril. D. Quetiapine. **9. Correct answer = C.** **10.** A patient whose angina was previously well controlled with once-daily isosorbide mononitrate states that recently he has been taking isosorbide mononitrate twice a day to control angina symptoms that are occurring more frequently during early morning hours. Which of the following is the best option for this patient? A. Continue once-daily administration of isosorbide mononitrate but advise the patient to take this medication in the evening. B. Advise continuation of isosorbide mononitrate twice daily for full 24-hour coverage of anginal symptoms. C. Switch to isosorbide dinitrate, as this has a longer duration of action than the mononitrate. D. Switch to nitroglycerin patch for consistent drug delivery and advise him to wear the patch around the clock. **10. Correct answer = A.** **11.** A 60-year-old man presents to his primary care physician with a complaint of severe chest pain when he walks uphill to his home in cold weather. The pain disappears when he rests. After evaluation and discussion of treatment options, a decision is made to treat him with nitroglycerin. Which of the following is a common **direct or reflex effect of nitroglycerin**? A. Decreased heart rate B. Decreased venous capacitance C. Increased afterload D. Increased cardiac force E. Increased diastolic myocardial fiber tension **11. The answer is D.** **12.** A 60-year-old man presents to his primary care physician with a complaint of severe chest pain when he walks uphill to his home in cold weather. The pain disappears when he rests. After evaluation and discussion of treatment options, a decision is made to treat him with nitroglycerin. In advising the patient about the adverse effects he may notice, you point out that nitroglycerin in moderate doses often produces certain symptoms. Which of the **following effects might occur due to the mechanism listed**? A. Apnea due to cranial vasodilation B. Dizziness due to reduced cardiac force of contraction C. Diuresis due to sympathetic discharge D. Headache due to meningeal vasodilation E. Hypertension due to reflex tachycardia **12. The answer is D.** **13.** Which medication should be prescribed to all anginal patients to treat an acute attack? A. Isosorbide dinitrate. B. Nitroglycerin patch. C. Nitroglycerin sublingual tablet or spray. D. Ranolazine. **13. Correct answer = C.** **14.** A 60-year-old man presents to his primary care physician with a complaint of severe chest pain when he walks uphill to his home in cold weather. The pain disappears when he rests. After evaluation and discussion of treatment options, a decision is made to treat him with nitroglycerin. **If a β blocker were to be used for prophylaxis in this patient,** what is the most probable mechanism of action in angina? A. Block of exercise-induced tachycardia B. Decreased end-diastolic ventricular volume C. Increased double product D. Increased cardiac force E. Decreased ventricular ejection time **14. The answer is A.** **15.** A new 60-year-old patient presents to the medical clinic with hypertension and angina. In considering adverse effects of possible drugs for these conditions, you note that an adverse effect that nitroglycerin and prazosin have in common is A. Bradycardia B. Impaired sexual function C. Lupus erythematosus syndrome D. Orthostatic hypotension E. Throbbing headache **15. The answer is D.** **16.** A 60-year-old woman had a myocardial infarction. Which of the following should be used to prevent life-**threatening arrhythmias that can occur post--myocardial infarction in this patient?** A. Digoxin. B. Flecainide. C. Metoprolol. D. Procainamide. E. Quinidine. **16. Correct answer = C.** A man is admitted to the emergency department with a brownish cyanotic appearance, marked shortness of breath, and hypotension. Which of the following is most likely to cause methemoglobinemia? A. Amyl nitrite B. Isosorbide dinitrate C. Isosorbide mononitrate D. Nitroglycerin E. Sodium cyanide ANS=A 17\. Another patient is admitted to the emergency department after a drug overdose. He is noted to have hypotension and severe tachycardia. He has been receiving therapy for hypertension and angina. Which of the following drugs often causes tachycardia? A. Clonidine B. Diltiazem C. Isosorbide dinitrate D. Propranolol E. Verapamil ANS=C 18\. A 45-year-old woman with hyperlipidemia and frequent migraine headaches develops angina of effort. Which of the following is relatively contraindicated because of her migraines? A. Amlodipine B. Diltiazem C. Metoprolol D. Nitroglycerin E. Verapamil ANS=D 19\. When nitrates are used in combination with other drugs for the treatment of angina, which one of the following combinations results in additive effects on the variable specified? A. Beta blockers and nitrates on end-diastolic cardiac size B. Beta blockers and nitrates on heart rate C. Calcium channel blockers and β blockers on cardiac force D. Beta blockers and nitrates on venous tone E. Calcium channel blockers and nitrates on heart rate ANS=C 20\. Certain drugs can cause severe hypotension when combined with nitrates. Which of the following interacts with nitroglycerin by inhibiting the metabolism of cGMP? A. Atenolol B. Hydralazine C. Isosorbide mononitrate D. Nifedipine E. Ranolazine F. Sildenafil G. Terbutaline ANS=A A 58-year-old woman who is obese presents to the emergency department with diaphoresis and crushing chest pain that radiates to her left arm. The physician orders an ECG and checks her cardiac enzymes to confirm his suspicion of myocardial infarction. Because of the quick response and intervention, she survives and is ultimately discharged with a prescription for low-dose daily aspirin to inhibit platelet aggregation. Two weeks after discharge, she takes ibuprofen for a tension headache. What is the effect of the ibuprofen on her anticoagulation regimen? \(A) Excessive antiplatelet activity because of a synergistic action between aspirin and ibuprofen on platelets \(B) Excessive antiplatelet activity because of ibuprofen's effects on endothelial cells combined with aspirin's effects on platelets \(C) Insufficient antiplatelet activity because ibuprofen induces liver cytochrome P450 metabolism of aspirin \(D) Insufficient antiplatelet activity because of inadequate platelet cyclooxygenase inhibition \(E) No change ANS=D **Antiarrhitmics** **17.** Suppression of arrhythmias resulting from a reentry focus is most likely to occur if the drug: A. Has vagomimetic effects on the AV node. B. Is a β-blocker. C. Converts a unidirectional block to a bidirectional block. D. Slows conduction through the atria. E. Has atropine-like effects on the AV node. **17. Correct answer = C.** **18.** A 57-year-old man is being treated for an atrial arrhythmia. He complains of **[dry mouth, blurred vision,]** and urinary hesitancy. Which antiarrhythmic drug is he mostly like taking? A. Metoprolol. B. Disopyramide. C. Dronedarone. D. Sotalol. **18. Correct answer = B.** **19.** A 58-year-old woman is being treated for chronic suppression of a ventricular arrhythmia. After 1 week of therapy, she complains about **[feeling severe upset stomach and heartburn]**. Which antiarrhythmic drug is the likely cause of these symptoms? A. Amiodarone. B. Digoxin. C. Mexiletine. D. Propranolol. E. Quinidine. **19. Correct answer = C**. **20.** A 78-year-old woman has been newly diagnosed with atrial fibrillation. She is not currently having **symptoms of palpitations or fatigue. Which is appropriate to initiate for rate control as an outpatient**? A. Amiodarone. B. Dronedarone. C. Esmolol. D. Flecainide. E. Metoprolol. **20. Correct answer = E.** **21.** A 76-year-old patient with rheumatoid arthritis and chronic heart disease is being considered for treatment with procainamide. She is already receiving digoxin, hydrochlorothiazide, and potassium supplements for her cardiac condition**. If this patient should take an overdose and manifest severe acute procainamide** toxicity with markedly prolonged QRS, which of the following should be given immediately? A. A calcium chelator such as EDTA B. Digitalis C. Nitroprusside D. Potassium chloride E. Sodium lactate **21. The answer is E.** **22.** All of the following are adverse effects of amiodarone except: A. Cinchonism. B. Hypothyroidism. C. Hyperthyroidism. D. Pulmonary fibrosis. E. Blue skin discoloration. **22. Correct answer = A.** **23.** Which arrhythmia can be treated **with lidocaine?** A. Paroxysmal supraventricular ventricular tachycardia. B. Atrial fibrillation. C. Atrial flutter. D. Ventricular tachycardia. **23. Correct answer = D.** **24.** A clinician wo665555uld like to initiate a drug for rhythm control of atrial fibrillation. Which of the following coexisting conditions would allow for initiation **[of flecainide]**? A. Hypertension. B. Left ventricular hypertrophy. C. Coronary artery disease. D. Heart failure. **24. Correct answer = A.** **25.** Which statement regarding dronedarone is correct? A. Dronedarone is more effective than amiodarone. B. QT interval prolongation is not a risk with dronedarone. C. Dronedarone increases the risk of death in patients with permanent atrial fibrillation or symptomatic heart failure. D. There is no need to monitor liver function with dronedarone. **25. Correct answer C.** **26.** A 76-year-old patient with rheumatoid arthritis and chronic heart disease is being considered for treatment with procainamide. **She is already receiving digoxin, hydrochlorothiazide, and potassium supplements for her cardiac condition. In deciding on a treatment regimen with procainamide** for this patient, which of the following statements is most correct? A. A possible drug interaction with digoxin suggests that digoxin blood levels should be obtained before and after starting procainamide B. Hyperkalemia should be avoided to reduce the likelihood of procainamide toxicity C. Procainamide cannot be used if the patient has asthma because it has a β-blocking effect D. Procainamide has a duration of action of 36--40 h E. Procainamide is not active by the oral route **26. The answer is B.** **27.** Which of the following is correct regarding digoxin when used for atrial fibrillation? A. Digoxin works by blocking voltage-sensitive calcium channels. B. Digoxin is used for rhythm control in patients with atrial fibrillation. C. Digoxin increases conduction velocity through the AV node. D. Digoxin levels of 1 to 2 ng/mL are desirable in the treatment of atrial fibrillation. **27 Correct answer = D.** **28.** A 57-year-old man is admitted to the emergency department with an irregular heart rate. The ECG shows an inferior myocardial infarction and ventricular tachycardia. Lidocaine is ordered. When used as an **antiarrhythmic drug, lidocaine typically** A. Increases action potential duration B. Increases contractility C. Increases PR interval D. Reduces abnormal automaticity E. Reduces resting potential **28. The answer is D.**. A 60-year-old woman had a myocardial infarction. Which of the following should be used to prevent life-threatening arrhythmias that can occur post--myocardial infarction in this patient? A. Digoxin. B. Flecainide. C. Metoprolol. D. Procainamide. E. Quinidine. ANS=C A 36-year-old woman with a history of poorly controlled thyrotoxicosis has recurrent episodes of tachycardia with severe shortness of breath. When she is admitted to the emergency department with one of these episodes, which of the following drugs would be most suitable? A. Amiodarone B. Disopyramide C. Esmolol D. Quinidine E. Verapamil ANS=C 15\. A 16-year-old girl has paroxysmal attacks of rapid heart rate with palpitations and shortness of breath. These episodes occasionally terminate spontaneously but often require a visit to the emergency department of the local hospital. Her ECG during these episodes reveals an AV nodal tachycardia. The antiarrhythmic of choice in most cases of acute AV nodal tachycardia is A. Adenosine B. Amiodarone C. Flecainide D. Propranolol E. Verapamil ANS=A 16\. A 55-year-old man is admitted to the emergency department and is found to have an abnormal ECG. Overdose of an antiarrhythmic drug is considered. Which of the following drugs is correctly paired with its ECG effects? A. Quinidine: Increased PR and decreased QT intervals B. Flecainide: Increased PR, QRS, and QT intervals C. Verapamil: Increased PR interval D. Lidocaine: Decreased QRS and PR interval E. Metoprolol: Increased QRS duration ANS=C 17\. A 60-year-old man comes to the emergency department with severe chest pain. ECG reveals ventricular tachycardia with occasional normal sinus beats, and ST-segment changes suggestive of ischemia. A diagnosis of myocardial infarction is made, and the man is admitted to the cardiac intensive care unit. His arrhythmia should be treated immediately with A. Adenosine B. Digoxin C. Lidocaine D. Quinidine E. Verapamil ANS=C 18\. Which of the following drugs slows conduction through the AV node and has its primary action directly on L-type calcium channels? A. Adenosine B. Amiodarone C. Diltiazem D. Esmolol E. Flecainide F. Lidocaine G. Mexiletine H. Procainamide I. Quinidine ANS=C 19\. When working in outlying fields, this 62-year-old farmer is away from his house for 12--14 h at a time. He has an arrhythmia that requires chronic therapy. Which of the following has the longest half-life of all antiarrhythmic drugs? A. Adenosine B. Amiodarone C. Disopyramide D. Esmolol E. Flecainide F. Lidocaine G. Mexiletine H. Procainamide I. Quinidine J. Verapamil ANS=B **Drugs affecting coagulation** **29.** Which of the P2Y12 ADP receptor antagonists reversibly binds the receptor? A. Clopidogrel. B. Prasugrel. C. Ticagrelor. D. Ticlopidine. **29. Correct answer = C.** **Monitoring** **30.** A 65-year-old man is brought to the emergency department 30 min after the onset of right-sided weakness and aphasia (difficulty speaking). Imaging studies ruled out cerebral hemorrhage as the cause of his acute symptoms of stroke. If the patient is unable to tolerate the Alteplase , he may be treated with clopidogrel. Relative to ticlopidine, clopidogrel A. Has a shorter duration of action B. Is less likely to cause neutropenia C. Is more likely to induce antiplatelet antibodies D. Is more likely to precipitate serious bleeding E. Will have a greater antiplatelet effect **30. The answer is B.** **31.** An 80-year-old male is taking warfarin indefinitely for the prevention of deep venous thrombosis. He is a compliant patient with a stable INR and has no issues with bleeding or bruising. He is diagnosed with a urinary tract infection and is prescribed sulfamethoxazole/trimethoprim. What effect will this have on his warfarin therapy? A. Sulfamethoxazole/trimethoprim will decrease the anticoagulant effect of warfarin. B. Sulfamethoxazole/trimethoprim will increase the anticoagulant effect of warfarin. C. Sulfamethoxazole/trimethoprim will activate platelet activity. D. Sulfamethoxazole/trimethoprim will not change anticoagulation status. **31. Correct answer = B.** **32.** In which disease state is cilostazol contraindicated? A. Peripheral arterial disease. B. Gout. C. Heart failure with reduced ejection fraction. D. Osteoporosis. **32. Correct answer = C.** **33.** Which must heparin bind to in order to exert its anticoagulant effect? A. GP IIb/IIIa receptor. B. Thrombin. C. Antithrombin III. D. von Willebrand factor. **33. Correct answer = C.** **34.** Which is considered "fibrin selective" because it rapidly activates plasminogen that is bound to fibrin? A. Alteplase. B. Fondaparinux. C. Streptokinase. D. Urokinase. **34. Correct answer = A.** **35.** A 58-year-old business executive is brought to the emergency department 2 h after the onset of severe chest pain during a vigorous tennis game. She has a history of poorly controlled mild hypertension and elevated blood cholesterol but does not smoke. ECG changes confirm the diagnosis of myocardial infarction. The decision is made to attempt to open her occluded artery. **If a fibrinolytic drug is used for treatment of this woman**'s acute myocardial infarction, which of the following adverse drug effects is most likely to occur? A. Acute renal failure B. Development of antiplatelet antibodies C. Encephalitis secondary to liver dysfunction D. Hemorrhagic stroke E. Neutropenia **35. The answer is D.** **36.** Which is most appropriate for reversing the anticoagulant effects of heparin? A. Aminocaproic acid. B. Protamine sulfate. C. Vitamin K1. D. Tranexamic acid. **36. Correct answer = B.** **37.** A 62-year-old male taking warfarin for stroke prevention in atrial fibrillation presents to his primary care physician with an elevated INR of 10.5 without bleeding. He is instructed to hold his warfarin dose and given 2.5 mg of oral vitamin K~1~. When would the effects of vitamin K on the INR most likely be noted in this patient? A. 1 hour. B. 6 hours. C. 24 hours. D. 72 hours. **37. Correct answer = C.** **38.** A 58-year-old man receives intravenous alteplase treatment for acute stroke. Five minutes following completion of alteplase infusion, he develops orolingual angioedema. Which of the following drugs may have increased the risk of developing orolingual angioedema in this patient? A. ACE inhibitor. B. GP IIb/IIIa receptor antagonist. C. Phosphodiesterase inhibitor. D. Thiazide diuretic. **38. Correct answer = A.** **39.** A 60-year-old business executive is brought to the emergency department 2 h after the onset of severe chest pain during a vigorous tennis game. She has a history of poorly controlled mild hypertension and elevated blood cholesterol but does not smoke. ECG changes confirm the diagnosis of myocardial infarction. The decision is made to attempt to open her occluded artery. Which of the following drugs accelerates the conversion of plasminogen to plasmin? A. Aminocaproic acid B. Heparin C. Lepirudin D. Reteplase E. Warfarin **39. The answer is D.** **40.** A 56-year-old man presents to the emergency room with complaints of swelling, redness, and pain in his right leg. The patient is diagnosed with acute DVT and requires treatment with an anticoagulant. All of the following are approved for **treatment of this patient's DVT except**: A. Rivaroxaban. B. Dabigatran. C. Enoxaparin. D. Heparin. **40. Correct answer = B.** **41.** A 65-year-old man is brought to the emergency department 30 min after the onset of right-sided weakness and aphasia (difficulty speaking). Imaging studies ruled out cerebral hemorrhage as the cause of his acute symptoms of stroke. Prompt administration of which of the following drugs is most likely to improve this patient's clinical outcome? A. Abciximab B. Alteplase C. Factor VIII D. Streptokinase E. Vitamin K **41. The answer is B.** **42.** A 70-year-old female is diagnosed with non-valvular atrial fibrillation. Her past medical history is significant for chronic kidney disease, and her renal function is moderately diminished. All of the following anticoagulants would be expected to require a reduced dosage in this patient except: A. Apixaban. B. Dabigatran. C. Rivaroxaban. D. Warfarin. **42. Correct answer = D.** **43.** A patient **develops severe thrombocytopenia in response to treatment with unfractionated heparin** and still requires parenteral anticoagulation. The patient is most likely to be treated with which of the following? A. Abciximab B. Cilostazol C. Lepirudin D. Plasminogen E. Vitamin K1 **43. The answer is C.** A 58-year-old business executive is brought to the emergency department 2 h after the onset of severe chest pain during a vigorous tennis game. She has a history of poorly controlled mild hypertension and elevated blood cholesterol but does not smoke. ECG changes confirm the diagnosis of myocardial infarction. The decision is made to attempt to open her occluded artery. Which of the following drugs accelerates the conversion of [plasminogen to plasmin]? A. Aminocaproic acid B. Heparin C. Lepirudin D. Reteplase E. Warfarin 12\. A 58-year-old business executive is brought to the emergency department 2 h after the onset of severe chest pain during a vigorous tennis game. She has a history of poorly controlled mild hypertension and elevated blood cholesterol but does not smoke. ECG changes confirm the diagnosis of myocardial infarction. The decision is made to attempt to open her occluded artery. If a fibrinolytic drug is used for treatment of this woman's acute myocardial infarction, which of the following adverse drug effects is most likely to occur? A. Acute renal failure B. Development of antiplatelet antibodies C. Encephalitis secondary to liver dysfunction D. Hemorrhagic stroke E. Neutropenia 13\. A 58-year-old business executive is brought to the emergency department 2 h after the onset of severe chest pain during a vigorous tennis game. She has a history of poorly controlled mild hypertension and elevated blood cholesterol but does not smoke. ECG changes confirm the diagnosis of myocardial infarction. The decision is made to attempt to open her occluded artery. If this patient undergoes a percutaneous coronary angiography procedure and placement of a stent in a coronary blood vessel, she may be given eptifibatide. Which of the following most accurately describes the mechanism of eptifibatide anticlotting action? A. Activation of antithrombin III B. Blockade of post-translational modification of clotting factors C. Inhibition of thromboxane production D. Irreversible inhibition of platelet ADP receptors E. Reversible inhibition of glycoprotein IIb/IIIa receptors 14\. A 65-year-old man is brought to the emergency department 30 min after the onset of right-sided weakness and aphasia (difficulty speaking). Imaging studies ruled out cerebral hemorrhage as the cause of his acute symptoms of stroke. Prompt administration of which of the following drugs is most likely to improve this patient's clinical outcome? A. Abciximab B. Alteplase C. Factor VIII D. Streptokinase E. Vitamin K 15\. A 65-year-old man is brought to the emergency department 30 min after the onset of right-sided weakness and aphasia (difficulty speaking). Imaging studies ruled out cerebral hemorrhage as the cause of his acute symptoms of stroke. Over the next 2 d, the patient's symptoms resolved completely. To prevent a recurrence of this disease, the patient is most likely to be treated indefinitely with which of the following? A. Aminocaproic acid B. Aspirin C. Enoxaparin D. Lepirudin E. Warfarin 16\. A 65-year-old man is brought to the emergency department 30 min after the onset of right-sided weakness and aphasia (difficulty speaking). Imaging studies ruled out cerebral hemorrhage as the cause of his acute symptoms of stroke. If the patient is unable to tolerate the Alteplase , he may be treated with clopidogrel. Relative to ticlopidine, clopidogrel A. Has a shorter duration of action B. Is less likely to cause neutropenia C. Is more likely to induce antiplatelet antibodies D. Is more likely to precipitate serious bleeding E. Will have a greater antiplatelet effect 17\. A 67-year-old woman presents with pain in her left thigh muscle. Duplex ultrasonography indicates the presence of deep vein thrombosis (DVT) in the affected limb. The decision was made to treat this woman with enoxaparin. Relative to unfractionated heparin, enoxaparin A. Can be used without monitoring the patient's aPTT B. Has a shorter duration of action C. Is less likely to have a teratogenic effect D. Is more likely to be given intravenously E. Is more likely to cause thrombosis and thrombocytopenia 18\. A 67-year-old woman presents with pain in her left thigh muscle. Duplex ultrasonography indicates the presence of deep vein thrombosis (DVT) in the affected limb. The decision was made to treat this woman with enoxaparin. During the next week, the patient was started on warfarin and her heparin was discontinued. Two months later, she returned after a severe nosebleed. Laboratory analysis revealed an INR (international normalized ratio) of 7.0 (INR value in such a warfarin-treated patient should be 2.0--3.0). To prevent severe hemorrhage, the warfarin should be discontinued and this patient should be treated immediately with which of the following? A. Aminocaproic acid B. Desmopressin C. Factor VIII D. Protamine E. Vitamin K1 19\. A patient develops severe thrombocytopenia in response to treatment with unfractionated heparin and still requires parenteral anticoagulation. The patient is most likely to be treated with which of the following? A. Abciximab B. Cilostazol C. Lepirudin D. Plasminogen E. Vitamin K1 **antiHypertension** **44.** A 45-year-old man was just started on therapy for hypertension and developed a **persistent, dry cough**. Which is most likely responsible for this side effect? A. Enalapril. B. Losartan. C. Nifedipine. D. Prazosin. E. Propranolol. **44**. **Correct answer = A.** **45.** Which may cause reflex tachycardia and/or postural hypotension on initial administration? A. Atenolol. B. Hydrochlorothiazide. C. Metoprolol. D. Prazosin. E. Verapamil. **45. Correct answer = D.** **46.** Which can precipitate a hypertensive crisis following **abrupt cessation of therapy**? A. Clonidine. B. Diltiazem. C. Enalapril. D. Losartan. E. Hydrochlorothiazide. **46. Correct answer = A.** **47.** A 48-year-old hypertensive patient has been successfully treated with a thiazide diuretic for the last 5 years. Over the last 3 months, his diastolic pressure has steadily increased, and he was started on an additional antihypertensive agent. He complains of several instances of **[being unable to achieve an erection and not being able]** to complete three sets of tennis as he once did. Which is the likely second antihypertensive medication? A. Captopril. B. Losartan. C. Metoprolol. D. Minoxidil. E. Nifedipine. **47. Correct answer = C.** **48.** A 40-year-old male has recently been diagnosed with hypertension due to pressure readings of 163/102 and 165/100 mm Hg. He also has diabetes that is well controlled with oral hypoglycemic medications. Which is the best **initial treatment regimen** for treatment of hypertension in this patient? A. Felodipine. B. Furosemide. C. Lisinopril. D. Lisinopril and hydrochlorothiazide. E. Metoprolol. **48. Correct answer = D.** **49.** A 60-year-old white female has not reached her blood pressure goal after 1 month of treatment with a low dose of lisinopril. All of the following would be appropriate next steps in the treatment of her hypertension except: A. Increase dose of lisinopril. B. Add a diuretic medication. C. Add on a calcium channel blocker medication. D. Add on an ARB medication. **49. Correct answer = D.** **50.** A patient returns to her health care provider for routine monitoring 3 months after her hypertension regimen was modified. **Labs reveal elevated serum potassium**. Which is likely responsible for this hyperkalemia? A. Chlorthalidone. B. Losartan. C. Furosemide. D. Clonidine. E. Nifedipine. **50. Correct answer = B.** **51.** A 58-year-old female reports that she recently stopped taking her blood pressure medications because of **swelling in her feet that began shortly** after she started treatment. Which is most likely **to cause peripheral edema**? A. Atenolol. B. Clonidine. C. Felodipine. D. Hydralazine. E. Prazosin. **51. Correct answer = C.** **52.** Which is an appropriate choice for hypertension ***treatment during pregnanc***y? A. Aliskiren. B. Fosinopril. C. Hydralazine. D. Valsartan. **52. Correct answer = C.** **53.** DD is a 50-year-old male with newly diagnosed hypertension. His comorbidities include diabetes and **[chronic hepatitis C infection with moderate liver impairment]**. He requires two drugs for initial treatment of his hypertension. Which should be prescribed in combination with a thiazide diuretic? A. Lisinopril. B. Spironolactone. C. Fosinopril. D. Furosemide. E. Hydralazine. **53. Correct answer = A.** **54.** A 32-year-old woman with hypertension wishes to become pregnant. Her physician informs her that she will have to switch to another antihypertensive drug. Which of the following drugs is absolutely **contraindicated in pregnancy**? A. Atenolol B. Captopril C. Methyldopa D. Prazosin E. Propranolol **54. The answer is B.** **55.** A patient is admitted to the emergency department with severe tachycardia after a drug overdose. His family reports that he has been depressed about his hypertension. Which one of the following drugs increases **[the heart rate in a dose-dependent manner]**? A. Captopril B. Hydrochlorothiazide C. Minoxidil D. Prazosin E. Verapamil **56. The answer is C.** **57.** Which one of the following is characteristic of captopril treatment in patients with essential hypertension? A. Competitively blocks angiotensin II at its receptor B. Decreases angiotensin II concentration in the blood C. Decreases renin concentration in the blood D. Increases sodium and decreases potassium in the blood E. Decreases sodium and increases potassium in the urine **57. The answer is B.** A 73-year-old man with a history of falling at home is found to have moderately severe hypertension. Which of the following drug groups is most likely to cause postural hypotension and thus an increased risk of falls? A. ACE inhibitors B. Alpha-receptor blockers C. Arteriolar dilators D. Beta1-selective receptor blockers E. Nonselective β blockers **The answer is B.** 15\. A significant number of patients started on ACE inhibitor therapy for hypertension are intolerant and must be switched to a different class of drug. What is the most common manifestation of this intolerance? A. Diarrhea B. Glaucoma C. Incessant cough D. Lupus-like syndrome E. Vomiting **The answer is C** 16\. Which one of the following is a significant unwanted effect of the drug named? A. Heart failure with hydralazine B. Hemolytic anemia with atenolol C. Fetal damage with losartan D. Lupus-like syndrome with hydrochlorothiazide E. Tachycardia with verapamil **The answer is C** 17\. Comparison of prazosin with atenolol shows that A. Both decrease heart rate B. Both increase cardiac output C. Both increase renin secretion D. Both increase sympathetic outflow from the CNS E. Both produce orthostatic hypotension **The answer is D** 18\. A patient with hypertension is to receive a calcium channel blocker. Verapamil is associated with which one of the following? A. Diarrhea B. Hypoglycemia C. Increased PR interval D. Tachycardia E. Thyrotoxicosis **The answer is C** 19\. A 45-year-old man is brought to the emergency department with mental obtundation. He is found to have a blood pressure of 220/160 and retinal hemorrhages. Which one of the following is used in severe hypertensive emergencies, is short-acting, acts on a G protein-coupled receptor, and must be given by intravenous infusion? A. Aliskiren B. Captopril C. Fenoldopam D. Hydralazine E. Losartan F. Metoprolol G. Nifedipine H. Prazosin I. Propranolol **The answer is C** 20\. Which of the following is very short-acting and acts by releasing nitric oxide? A. Atenolol B. Captopril C. Diltiazem D. Fenoldopam E. Hydrochlorothiazide F. Losartan G. Minoxidil H. Nitroprusside I. Prazosin **The answer is H** **Diuretics** **58.** An elderly patient with a history of heart disease is brought to the emergency room with difficulty breathing. Examination reveals **[that she has pulmonary edema]**. Which treatment is indicated? A. Acetazolamide. B. Chlorthalidone. C. Furosemide. D. Hydrochlorothiazide. E. Spironolactone. **58. Correct answer = C.** **59.** A group of college students is planning a mountain climbing trip to the Andes. Which would be appropriate for them to take to prevent mountain sickness? A. A thiazide diuretic such as hydrochlorothiazide. B. An anticholinergic such as atropine. C. A carbonic anhydrase inhibitor such as acetazolamide. D. A loop diuretic such as furosemide. E. A β-blocker such as metoprolol. **59. Correct answer = C.** **60.** An alcoholic male has **developed hepatic cirrhosis**. To control the ascites and edema, which should be prescribed? A. Acetazolamide. B. Chlorthalidone. C. Furosemide. D. Hydrochlorothiazide. E. Spironolactone. **60. Correct answer = E.** **62.** A 55-year-old male with kidney stones has been placed on a diuretic to decrease calcium excretion. However, after a few weeks, he develops an attack of gout. Which diuretic was he taking? A. Furosemide. B. Hydrochlorothiazide. C. Spironolactone. D. Triamterene. E. Urea. **62. Correct answer = B.** **63.** A 75-year-old woman with hypertension is being treated with a thiazide. Her blood pressure responds well and reads at 120/76 mm Hg. After several months on the medication, she complains of being tired and weak. An analysis of the blood indicates low values for which of the following? A. Calcium. B. Glucose. C. Potassium. D. Sodium. E. Uric acid. **63. Correct answer = C.** **64.** Which is contraindicated in a patient with hyperkalemia? A. Acetazolamide. B. Chlorthalidone. C. Chlorothiazide. D. Ethacrynic acid. E. Spironolactone. **64. Correct answer = E.** **65.** Which of the following should be avoided in a patient with a history of severe **anaphylactic reaction** to **sulfa medications**? A. Amiloride. B. Hydrochlorothiazide. C. Mannitol. D. Spironolactone. E. Triamterene. **65. Correct answer = B.** **66.** A male patient is placed on a new medication and notes that his **breasts have become enlarged** and tender to the touch. Which medication is he most likely taking? A. Chlorthalidone. B. Furosemide. C. Hydrochlorothiazide. D. Spironolactone. E. Triamterene. **66. Correct answer = D.** **68.** A patient presents to the emergency department with an extreme headache. After a thorough workup, the attending physician concludes that the pain is due to **increased intracranial pressure**. Which diuretic would work best to reduce this pressure? A. Acetazolamide. B. Indapamide. C. Furosemide. D. Hydrochlorothiazide. E. Mannitol. **68. Correct answer = E.** **69.** Which diuretic has been shown to **improve blood pressure in resistant hypertension** or those already treated with three blood pressure medications including a thiazide or thiazide-like diuretic? A. Chlorthalidone. B. Indapamide. C. Furosemide. D. Mannitol. E. Spironolactone. **69. Correct answer = E.** **70.** A 70-year-old retired businessman is admitted with a history of recurrent heart failure and metabolic derangements. He has **[marked peripheral edema and metabolic alkalosis]**. Which of the following drugs is most appropriate for the treatment of his edema? A. Acetazolamide B. Digoxin C. Dobutamine D. Hydrochlorothiazide E. Spironolactone **70.** **The answer is A.** **71.** A 50-year-old man has a history of frequent episodes of renal colic with calcium-containing renal stones. A careful workup indicates that he has a defect in proximal tubular calcium reabsorption, which results in high concentrations of calcium salts in the tubular urine. The most useful diuretic agent in **the treatment of recurrent calcium stones is** A. Acetazolamide B. Furosemide C. Hydrochlorothiazide D. Mannitol E. Spironolactone **71. The answer is C** **72.** Which of the following is an important effect of **chronic therapy with loop diuretics?** A. Decreased urinary excretion of calcium B. Elevation of blood pressure C. Elevation of pulmonary vascular pressure D. Metabolic acidosis E. Ototoxicity **.** **72. The answer is E.** Which drug is correctly associated with its actions in the following table? (+ indicates increase and -- indicates decrease.) Choice Drug Urine Na+ Urine K+ Metabolic change A Acetazolamide +++ + Alkalosis B Furosemide ++ - Alkalosis C Hydrochlorothiazide + ++ Acidosis D Spironolactone + - Acidosis E Mannitol - ++ Alkalosis 15\. Which of the following diuretics would be most useful in the acute treatment of a comatose patient with brain injury and cerebral edema? A. Acetazolamide B. Amiloride C. Ethacrynic acid D. Furosemide E. Mannitol 16\. A 62-year-old man with advanced prostate cancer is admitted to the emergency department with mental obtundation. An electrolyte panel shows a serum calcium of 16.5 (normal \~8.5--10.5 mg/dL). Which of the following therapies would be most useful in the management of severe hypercalcemia? A. Acetazolamide plus saline infusion B. Furosemide plus saline infusion C. Hydrochlorothiazide plus saline infusion D. Mannitol plus saline infusion E. Spironolactone plus saline infusion 17\. A 60-year-old patient complains of paresthesias and occasional nausea associated with one of her drugs. She is found to have hyperchloremic metabolic acidosis. She is probably taking A. Acetazolamide for glaucoma B. Amiloride for edema associated with aldosteronism C. Furosemide for severe hypertension and heart failure D. Hydrochlorothiazide for hypertension E. Mannitol for cerebral edema 18\. A 70-year-old woman is admitted to the emergency department because of a "fainting spell" at home. She appears to have suffered no trauma from her fall, but her blood pressure is 120/60 when lying down and 60/20 when she sits up. Neurologic examination and an ECG are within normal limits when she is lying down. Questioning reveals that she has recently started taking "water pills" (diuretics) for a heart condition. Which of the following drugs is the most likely cause of her fainting spell? A. Acetazolamide B. Amiloride C. Furosemide D. Hydrochlorothiazide E. Spironolactone 19\. A 58-year-old woman with lung cancer has abnormally low serum osmolality. A drug that increases the formation of dilute urine and is used to treat SIADH is A. Acetazolamide B. Amiloride C. Conivaptan D. Desmopressin E. Ethacrynic acid F. Furosemide G. Hydrochlorothiazide H. Mannitol I. Spironolactone J. Triamterene 20\. A graduate student is planning to make a high-altitude climb in South America while on vacation. He will not have time to acclimate slowly to altitude. A drug that is useful in preventing high-altitude sickness is A. Acetazolamide B. Amiloride C. Demeclocycline D. Desmopressin E. Ethacrynic acid **Heart failure** **73.** Which drug may exacerbate HF? A. Acetaminophen. B. Cetirizine. C. Chlorthalidone. D. Ibuprofen. **73. Correct answer = D.** **74.** Which best describes the action of ACE inhibitors on the failing heart? A. ACE inhibitors increase vascular resistance. B. ACE inhibitors decrease cardiac output. C. ACE inhibitors reduce preload. D. ACE inhibitors increase aldosterone. **74. Correct answer = C.** **75.** What makes losartan different from other ARBs? A. Losartan is renally eliminated. B. Losartan has an active metabolite. C. Losartan has the shortest half-life. D. Losartan has a small volume of distribution. **75. Correct answer = B**. **76.** How do β-blockers improve cardiac function in HF? A. By decreasing cardiac remodeling. B. By increasing heart rate. C. By increasing renin release. D. By activating norepinephrine. **76. Correct answer = A**. **77.** BC is a 70-year-old female who is diagnosed with HFrEF. Her past medical history is significant for hypertension and atrial fibrillation. She is taking hydrochlorothiazide, lisinopril, metoprolol tartrate, and warfarin. BC says she is feeling "good" and has no cough, shortness of breath, or edema. Which is the most appropriate medication change to make? A. Discontinue hydrochlorothiazide. B. Change lisinopril to losartan. C. Decrease warfarin dose. D. Change metoprolol tartrate to metoprolol succinate. **77. Correct answer = D.** **78.** SC is a 75-year-old white male who has HF. He is seen in clinic today, reporting shortness of breath, **increased pitting edema, and a 5-pound weight gain over the last 2 days. His current medication regimen includes losartan and metoprolol succinate**. SC has no chest pain and is deemed stable for outpatient treatment. Which of the following is the best recommendation? A. Increase the dose of metoprolol succinate. B. Start hydrochlorothiazide. C. Start furosemide. D. Discontinue losartan. **78. Correct answer = C.** **79.** How is spironolactone beneficial in HF? A. Promotes potassium secretion. B. Agonizes aldosterone. C. Prevents cardiac hypertrophy. D. Decreases blood glucose. **79. Correct answer = C.** **80.** Which is important to monitor in patients taking digoxin? A. Chloride. B. Potassium. C. Sodium. D. Zinc. **80. Correct answer = B**. **81.** Which describes the mechanism of action **of milrinone in HF**? A. Decreases intracellular calcium. B. Increases cardiac contractility. C. Decreases cAMP. D. Activates phosphodiesterase. **81. Correct answer = B.** **82.** What is the most common adverse effect associated with fixed-dose hydralazine/isosorbide dinitrate? A. Diarrhea. B. Drug-induced lupus. C. Headache. D. Heartburn. **82. Correct answer = C.** **83.** A 73-year-old man with an inadequate response to other drugs is to receive digoxin for heart failure. Which of the following is the best**-documented mechanism of beneficial action of cardiac glycosides**? \(A) A decrease in calcium uptake by the sarcoplasmic reticulum \(B) An increase in ATP synthesis \(C) A modification of the actin molecule \(D) An increase in systolic cytoplasmic calcium levels \(E) A block of cardiac β adrenoceptors **83. The answer is D.** **84.** A 73-year-old man with an inadequate response to other drugs is to receive digoxin for heart failure. After your patient has been receiving digoxin for 3 wk, he presents to the emergency department with an arrhythmia. Which one of the following is most likely to **contribute to the arrhythmogenic effect of digoxin**? A. Increased parasympathetic discharge B. Increased intracellular calcium C. Decreased sympathetic discharge D. Decreased intracellular ATP E. Increased extracellular potassium **84. The answer is B.** **85.** A patient who has been taking digoxin for several years for atrial fibrillation and chronic heart failure is about to receive atropine for another condition. A common effect of digoxin (at therapeutic blood levels) that can be almost entirely blocked by atropine is A. Decreased appetite B. Headaches C. Increased atrial contractility D. Increased PR interval on ECG E. Tachycardia **85. The answer is D.** A 65-year-old woman has been admitted to the coronary care unit with a left ventricular myocardial infarction. She develops acute severe heart failure with marked pulmonary edema, but no evidence of peripheral edema or weight gain. Which one of the following drugs would be most useful? A. Digoxin B. Furosemide C. Minoxidil D. Propranolol E. Spironolactone 15\. An 82-year-old woman has long-standing heart failure. Which one of the following drugs has been shown to reduce mortality in chronic heart failure? A. Atenolol B. Digoxin C. Dobutamine D. Furosemide E. Spironolactone 16\. Which row in the following table correctly shows the major effects of full therapeutic doses of digoxin on the AV node and the ECG? Row Row AV Refractory Period QT Interval T Wave A Increased Increased Upright B Increased Decreased Inverted C Decreased Increased Upright D Decreased Decreased Upright E Decreased Increased Inverted 17\. Which one of the following drugs is associated with clinically useful or physiologically important positive inotropic effect? A. Captopril B. Dobutamine C. Enalapril D. Losartan E. Nesiritide 18\. A 38-year-old man who has been running a marathon collapses and is brought to the emergency department. He is found to have a left ventricular myocardial infarction and heart failure with significant pulmonary edema. The first-line drug of choice in most cases of heart failure is A. Atenolol B. Captopril C. Carvedilol D. Digoxin E. Diltiazem F. Dobutamine G. Enalapril H. Furosemide I. Metoprolol J. Spironolactone 19\. Which of the following has been shown to prolong life in patients with chronic congestive failure in spite of having a negative inotropic effect on cardiac contractility? A. Carvedilol B. Digoxin C. Dobutamine D. Enalapril E. Furosemide 20\. A 5-year-old child is brought to the emergency department with sinus arrest and a ventricular rate of 35 bpm. An empty bottle of his uncle's digoxin was found where he was playing. Which of the following is the drug of choice in treating a severe overdose of digoxin? A. Digoxin antibodies B. Lidocaine infusion C. Magnesium infusion D. Phenytoin by mouth E. Potassium by mouth **Hyperlipidemia** **86.** Which one of the following is the most common side effect of antihyperlipidemic drug therapy? A. Elevated blood pressure. B. Gastrointestinal disturbance. C. Neurologic problems. D. Heart palpitations. E. Migraine headaches. **86. Correct answer = B.** **87.** Which one of the following hyperlipidemias is characterized by elevated plasma levels of chylomicrons and has no drug therapy available to lower the plasma lipoprotein levels? A. Type I. B. Type II. C. Type III. D. Type IV. E. Type V. **87. Correct answer = A.** **88.** Which one of the following drugs decreases cholesterol synthesis by inhibiting the enzyme 3-hydroxy-3- **methylglutaryl coenzyme A reductase**? A. Fenofibrate. B. Niacin. C. Cholestyramine. D. Lovastatin. E. Gemfibrozil. **88. Correct answer = D.** **89.** Which one of the following drugs causes a decrease in liver triglyceride synthesis by limiting available free fatty acids needed as **building blocks for this pathway**? A. Niacin. B. Fenofibrate. C. Cholestyramine. D. Gemfibrozil. E. Lovastatin. **89. Correct answer = A.** **90.** Which one of the following **drugs binds bile acids in the intestin**e, thus preventing their return to the liver via the enterohepatic circulation? A. Niacin. B. Fenofibrate. C. Cholestyramine. D. Fluvastatin. E. Lovastatin. **90. Correct answer = C.** **91.** JS is a 65-year-old man who presents to his physician for management of hyperlipidemia. His most recent lipid panel reveals an LDL cholesterol level of 165 mg/dL. His physician wishes to begin treatment to lower his LDL cholesterol levels. Which of the following therapies is the **best option to lower JS's LDL cholesterol levels?** A. Fenofibrate. B. Colesevelam. C. Niacin. D. Simvastatin. E. Ezetimibe. **91. Correct answer = D.** **92.** WW is a 62-year-old female with hyperlipidemia and hypothyroidism. Her current medications include cholestyramine and levothyroxine (thyroid hormone). What advice would you give to WW to avoid a drug interaction between her cholestyramine and levothyroxine? A. Stop taking the levothyroxine as it can interact with cholestyramine. B. Take levothyroxine 1 hour before cholestyramine on an empty stomach. C. Switch cholestyramine to colesevelam as this will eliminate the interaction. D. Switch cholestyramine to colestipol as this will eliminate the interaction. E. Take levothyroxine and cholestyramine at the same time to minimize the interaction. **92. Correct answer = B.** **93.** AJ is a 42-year-old man who was started on niacin sustained-release tablets 2 weeks ago for elevated triglycerides and low HDL levels. He is complaining of an uncomfortable flushing and itchy feeling that he thinks is related to the niacin. Which of the following options can help AJ manage this adverse effect of niacin therapy? A. Administer aspirin 30 minutes prior to taking niacin. B. Administer aspirin 30 minutes after taking niacin. C. Increase the dose of niacin SR to 1000 mg. D. Continue the current dose of niacin. E. Change the sustained-release niacin to immediate-release niacin. **93. Correct answer = A.** **94.** CN is a 72-year-old male who is treated for hyperlipidemia with high-dose atorvastatin for the past 6 months. He also has a history of renal insufficiency. His most recent lipid panel shows an LDL cholesterol level of 131 mg/dL, **triglycerides of 510 mg/dL, and HDL cholesterol** of 32 mg/dL. His physician wishes to add an additional agent for his hyperlipidemia. Which of the following choices is the best option to address CN's dyslipidemia? **94. Correct answer = B.** **95.** Which of the following patient populations is more **likely to experience myalgia (muscle pain)** or myopathy with use of HMG CoA reductase inhibitors? A. Patients with diabetes mellitus. B. Patients with renal insufficiency. C. Patients with gout. D. Patients with hypertriglyceridemia. E. Patients taking warfarin (blood thinner). **95. Correct answer = B.** **96.** Increased serum levels of which of the following is associated with a **decreased risk of atherosclerosis**? A. Cholesterol B. LDL C. HDL D. Triglyceride E. VLDL **96. The answer is C.** **97.** A 58-year-old man with a history of hyperlipidemia was treated with a drug. **The chart below shows the results of the patient's fasting** lipid panel before treatment and 6 mo after initiating drug therapy. Normal values are also shown. Which of the following drugs is most likely to be the one that this man received? A. Colestipol B. Ezetimibe C. Gemfibrozil D. Lovastatin E. Niacin **97. The answer is E.** **98.** A 35-year-old woman appears to have familial combined hyperlipidemia. Her serum concentrations of total cholesterol, LDL cholesterol, and triglyceride are elevated. Her serum concentration of HDL cholesterol is somewhat reduced. Which of the following drugs is most likely to increase this patient's triglyceride and VLDL **cholesterol concentrations when used as monotherapy**? A. Atorvastatin B. Cholestyramine C. Ezetimibe D. Gemfibrozil E. Niacin **98. The answer is B.** A 35-year-old woman appears to have familial combined hyperlipidemia. Her serum concentrations of total cholesterol, LDL cholesterol, and triglyceride are elevated. Her serum concentration of HDL cholesterol is somewhat reduced. The patient is started on gemfibrozil. Which of the following is a major mechanism of gemfibrozil's action? A. Increased excretion of bile acid salts B. Increased expression of high-affinity LDL receptors C. Increased secretion of VLDL by the liver D. Increased triglyceride hydrolysis by lipoprotein lipase E. Reduced uptake of dietary cholesterol 16\. Which of the following is a major toxicity associated with gemfibrozil therapy? A. Bloating and constipation B. Cholelithiasis C. Hyperuricemia D. Liver damage E. Severe cardiac arrhythmia 17\. A 43-year-old man has heterozygous familial hypercholesterolemia. His serum concentrations of total cholesterol and LDL are markedly elevated. His serum concentration of HDL cholesterol, VLDL cholesterol, and triglycerides are normal or slightly elevated. The patient's mother and older brother died of myocardial infarctions before the age of 50. This patient recently experienced mild chest pain when walking upstairs and has been diagnosed as having angina of effort. The patient is somewhat overweight. He drinks alcohol most evenings and smokes about 1 pack of cigarettes per week. Consumption of alcohol is associated with which of the following changes in serum lipid concentrations? A. Decreased chylomicrons B. Decreased HDL cholesterol C. Decreased VLDL cholesterol D. Increased LDL cholesterol E. Increased triglyceride 18\. A 43-year-old man has heterozygous familial hypercholesterolemia. His serum concentrations of total cholesterol and LDL are markedly elevated. His serum concentration of HDL cholesterol, VLDL cholesterol, and triglycerides are normal or slightly elevated. The patient's mother and older brotherdied of myocardial infarctions before the age of 50. This patient recently experienced mild chest pain when walking upstairs and has been diagnosed as having angina of effort. The patient is somewhat overweight. He drinks alcohol most evenings and smokes about 1 pack of cigarettes per week. If the patient has a history of gout, which of the following drugs is most likely to exacerbate this condition? A. Colestipol B. Ezetimibe C. Gemfibrozil D. Niacin E. Simvastatin 19\. A 43-year-old man has heterozygous familial hypercholesterolemia. His serum concentrations of total cholesterol and LDL are markedly elevated. His serum concentration of HDL cholesterol, VLDL cholesterol, and triglycerides are normal or slightly elevated. The patient's mother and older brother died of myocardial infarctions before the age of 50. This patient recently experienced mild chest pain when walking upstairs and has been diagnosed as having angina of effort. The patient is somewhat overweight. He drinks alcohol most evenings and smokes about 1 pack of cigarettes per week. After being counseled about lifestyle and dietary changes, the patient was started on atorvastatin. During his treatment with atorvastatin, it is important to routinely monitor serum concentrations of which of the following? A. Blood urea nitrogen B. Alanine and aspartate aminotransferase C. Platelets D. Red blood cells E. Uric acid 20\. A 43-year-old man has heterozygous familial hypercholesterolemia. His serum concentrations of total cholesterol and LDL are markedly elevated. His serum concentration of HDL cholesterol, VLDL cholesterol, and triglycerides are normal or slightly elevated. The patient's mother and older brother died of myocardial infarctions before the age of 50. This patient recently experienced mild chest pain when walking upstairs and has been diagnosed as having angina of effort. The patient is somewhat overweight. He drinks alcohol most evenings and smokes about 1 pack of cigarettes per week. Six months after beginning atorvastatin, the patient's total and LDL cholesterol concentrations remained above normal, and he continued to have anginal attacks despite good adherence to his antianginal medications. His physician decided to add ezetimibe. Which of the following is the most accurate description of ezetimibe's mechanism of an action? A. Decreased lipid synthesis in adipose tissue B. Decreased secretion of VLDL by the liver C. Decreased gastrointestinal absorption of cholesterol D. Increased endocytosis of HDL by the liver E. Increased lipid hydrolysis by lipoprotein lipase A 35-year-old woman appears to have familial combined hyperlipidemia. Her serum concentrations of total cholesterol, LDL cholesterol, and triglyceride are elevated. Her serum concentration of HDL cholesterol is somewhat reduced. If this patient is pregnant, which of the following drugs should be avoided because of a risk of harming the fetus? A. Cholestyramine B. Ezetimibe C. Fenofibrate D. Niacin E. Pravastatin **Sedatives** **1.** Which one of the following statements is **correct regarding benzodiazepines**? A. Benzodiazepines directly open chloride channels. B. Benzodiazepines show analgesic actions. C. Clinical improvement of anxiety requires 2 to 4 weeks of treatment with benzodiazepines. D. All benzodiazepines have some sedative effects. E. Benzodiazepines, like other CNS depressants, readily produce general anesthesia. **1. Correct answer = D.** **2.** Which one of the following is a **short-acting hypnotic**? A. Phenobarbital. B. Diazepam. C. Chlordiazepoxide. D. Triazolam. E. Flurazepam. **2. Correct answer = D.** **3.** Which one of the following statements is correct regarding the anxiolytic and hypnotic agents? A. Phenobarbital shows analgesic properties. B. Diazepam and phenobarbital induce the cytochrome P450 enzyme system. C. Phenobarbital is useful in the treatment of acute intermittent porphyria. D. Phenobarbital induces respiratory depression, which is enhanced by the consumption of ethanol. E. Buspirone has actions similar to those of the benzodiazepines. **3. Correct answer = D.** **4.** A 45-year-old man who has been injured in a car accident is brought into the emergency room. His blood alcohol level on admission is 275 mg/dL. Hospital records show a prior hospitalization for alcohol-related seizures. **His wife confirms that he has been drinking heavily for 3 weeks**. What treatment should be provided to the patient if he goes into withdrawal? A. None. B. Lorazepam. C. Pentobarbital. D. Phenytoin. E. Buspirone. **4. Correct answer = B.** **5.** Which one of the following is a short-acting hypnotic **[and better for sleep induction compared]** to sleep maintenance? A. Temazepam. B. Flurazepam. C. Zaleplon. D. Buspirone. E. Escitalopram. **5. Correct answer = C.** **6.** Which of the following agents has a rapid anxiolytic effect and would be best for **the acute management of anxiety?** A. Buspirone. B. Venlafaxine. C. Lorazepam. D. Escitalopram. E. Duloxetine. **6. Correct answer = C.** **7.** Which of the following sedative--hypnotic agents utilizes melatonin receptor agonism as the mechanism of action to induce sleep? A. Zolpidem. B. Eszopiclone. C. Estazolam. D. Ramelteon. E. Diphenhydramine. **7. Correct answer = D.** **8.** All of the following agents for the management of insomnia are controlled substances and may have a risk for **addiction or dependence except:** A. Zaleplon. B. Flurazepam. C. Doxepin. D. Zolpidem. E. Triazolam. **8. Correct answer = C.** **Antisizures** **1**. A 9-year-old boy is sent for neurologic evaluation because of episodes of apparent inattention. Over the past year, the child has experienced episodes during which he develops a **blank look on his face and his eyes blink for 15 seconds**. He immediately resumes his previous activity. Which one the following best describes this patient's seizures? A. Absence. B. Complex partial. C. Tonic--clonic. D. Simple partial. E. Myoclonic. **1. Correct answer = A.** **2**. A child is experiencing absence seizures that interrupt his ability to **pay attention during school** and activities. Which of the following therapies would be most appropriate for this patient? A. Ethosuximide. B. Carbamazepine. C. Diazepam. D. Carbamazepine plus primidone. E. Watchful waiting. **2. Correct answer = A.** **3**. Which of the following drugs is most useful for the **[treatment of absence seizures?]** A. Topiramate. B. Tiagabine. C. Levetiracetam. D. Lamotrigine. E. Zonisamide. **3. Correct answer = D.** **4.** A 25-year-old woman with myoclonic seizures is well controlled on valproate. She indicates that she is interested in **becoming pregnant in the next year.** With respect to her antiepilepsy medication, which of the following should be considered? A. Leave her on her current therapy. **B. Consider switching to lamotrigine.** C. Consider adding a second antiepilepsy medication. D. Decrease her valproate dose. **4. Correct answer = B.** **5**. A woman with myoclonic seizures is well controlled with lamotrigine. She becomes pregnant and begins to have breakthrough seizures. What is most likely happening? A. Her epilepsy is getting worse. B. Lamotrigine concentrations are increasing. **C. Lamotrigine concentrations are decreasing.** D. Lamotrigine is no longer efficacious for this patient. **5. Correct answer = C.** **6**. A 42-year-old man undergoes a neurologic evaluation because of episodes of apparent confusion. Over the past year, the man has experienced episodes during which he develops **a blank look on his face** and fails to respond to questions. Moreover, it **appears to take several minutes before** the man recovers from the episodes. Which one of the following best describes this type of seizure? A. Focal (simple partial). B. Focal (complex partial). C. Tonic--clonic. D. Absence. E. Myoclonic. **6. Correct answer = B.** **7.** A 52-year-old man has had several **focal complex partial seizures over the last year**. Which one of the following therapies would be the most appropriate initial therapy for this patient? A. Ethosuximide. B. Levetiracetam. C. Diazepam. D. Carbamazepine plus primidone. E. Watchful waiting. **7. Correct answer = B.** **8**. A patient with focal complex partial seizures has been treated for 6 months with carbamazepine but, recently, has been experiencing breakthrough seizures on a more frequent basis. You are considering adding a second drug to the antiseizure regimen. Which of the following drugs **is least likely to have a pharmacokinetic interaction with carbamazepine?** A. Topiramate. B. Tiagabine. C. Levetiracetam. D. Lamotrigine. E. Zonisamide. **8. Correct answer = C.** **Parkinson** 1\. Which one of the following combinations of antiparkinsonian drugs is an appropriate treatment plan? A. Amantadine, carbidopa, and entacapone. B. Levodopa, carbidopa, and entacapone. C. Pramipexole, carbidopa, and entacapone. D. Ropinirole, selegiline, and entacapone. E. Ropinirole, carbidopa, and selegiline. **1. Correct answer = B.** **2**. Peripheral adverse effects of levodopa, including nausea, hypotension, and cardiac arrhythmias, can be diminished by including which of the following drugs in the therapy? A. Amantadine. B. Ropinirole. C. Carbidopa. D. Tolcapone. E. Pramipexole. **2. Correct answer = C.** **3**. Which of the following antiparkinsonian drugs may cause vasospasm? A. Amantadine. B. Bromocriptine. C. Carbidopa. D. Entacapone. E. Ropinirole. **3. Correct answer = B.** **4**. Modest improvement in the memory of patients with Alzheimer's disease may occur with drugs that increase transmission at which of the following receptors? A. Adrenergic. B. Cholinergic. C. Dopaminergic. D. GABAergic. E. Serotonergic. **4. Correct answer = B.** **5**. Which medication is a glutamate receptor antagonist that can be used in **combination with an acetylcholinesterase inhibito**r to manage the symptoms of Alzheimer's disease? A. Rivastigmine. B. Ropinirole. C. Fluoxetine. D. Memantine. E. Donepezil. **5. Correct answer = D.** **6.** Which of the following agents is available as a patch for once-daily use and is likely to provide steady drug levels to treat Alzheimer's disease? A. Rivastigmine. B. Donepezil. C. Memantine. D. Galantamine. E. Glatiramer. **6. Correct answer = A.** **7**. Which of the following is the only medication that is approved for the management of **amyotrophic lateral sclerosis?** A. Pramipexole. B. Selegiline. C. Galantamine. D. Riluzole. E. Glatiramer. **7. Correct answer = D.** **8**. Which of the following medications reduces immune system--mediated inflammation via inhibition of **pyrimidine synthesis to reduce the number of activated lymphocytes in the CNS**? A. Riluzole. B. Rotigotine. C. Teriflunomide. D. Dexamethasone. **8. Correct answer = C.** **Opioids** **1.** A young woman is brought into the emergency room. She is unconscious, and she has pupillary constriction and depressed respiration. Based on reports, an opioid overdose is almost certain. Which of the listed phenanthrene opioids will exhibit a full and immediate response to treatment with naloxone? A. Meperidine. B. Morphine. C. Buprenorphine. D. Fentanyl. **1. Correct answer = B.** **2.** A 76-year-old female with renal insufficiency presents to the clinic with severe pain secondary to a compression fracture in the lumbar spine. She reports that the pain **has been uncontrolled with tramadol**, and it is decided to start treatment with an opioid. Which of the following is the best opioid for this patient? A. Meperidine. B. Fentanyl transdermal patch. C. Hydrocodone. D. Morphine. **2. Correct answer = C.** **3**. Which of the following statements about fentanyl is correct? A. Fentanyl is 100 times more potent than morphine. B. Its withdrawal symptoms can be relieved by naloxone. C. The active metabolites of fentanyl can cause seizures. D. It is most effective by oral administration. **3. Correct answer = A.** **4.** A 56-year-old patient who has suffered with severe chronic pain with radiculopathy secondary to spinal stenosis for years presents to the clinic for pain management. Over the years, this patient has failed to receive relief **from** the neuropathic pain from the radiculopathy with traditional agents such as tricyclics or anticonvulsants. Based on the mechanism of action, which opioid might be beneficial in this patient to treat **[both nociceptive and neuropathic pain]**? A. Meperidine. B. Oxymorphone. C. Morphine. D. Methadone. **4. Correct answer = D.** **5.** Which of the following statements regarding methadone is correct? **5. Correct Answer = C.** **6.** Which of the following opioids is the LEAST lipophilic? **6. Correct answer = D.** **7.** A 64-year-old male is preparing for a total knee replacement. He is taking many medications that are metabolized by the CYP450 enzyme system and is worried about drug interactions with the pain medication that will be used following his surgery. Which of the following opioids would have the lowest chance of interacting with his **medications that are metabolized by the CYP450 enzyme system?** A. Methadone. B. Oxymorphone. C. Oxycodone. D. Hydrocodone. **7. Correct answer = B.** **8.** Which of the following opioids is the best choice for treating pain associated with **diabetic peripheral neuropathy?** A. Morphine. B. Tapentadol. C. Codeine. D. Buprenorphine. **8. Correct answer = B.** **Antimicrobials cell wall inhibitors** **99**. A 45-year-old male presented to the hospital 3 days ago with severe cellulitis and a large abscess on his left leg. Incision and drainage were performed on the abscess, and cultures revealed methicillin-resistant Staphylococcus aureus. Which of the following would be the most appropriate treatment option for once daily outpatient intravenous therapy? **A.** Ertapenem. **B.** Ceftaroline. **C.** Daptomycin. **D.** Piperacillin/tazobactam. **99. Correct answer = C.** **100.** Which of the following adverse effects is associated with daptomycin? A. Ototoxicity. B. Red man syndrome. C. QT-prolongation. D. Rhabdomyolysis. **100. Correct answer = D.** **101**. A 72-year-old male is admitted to the hospital from a nursing home with severe pneumonia. He was recently discharged from the hospital 1 week ago after open heart surgery. The patient has no known allergies. Which of the following regimens is most appropriate for empiric coverage of methicillinresistant Staphylococcus aureus and Pseudomonas aeruginosa in this patient? A. Vancomycin + cefepime + ciprofloxacin. B. Vancomycin + cefazolin + ciprofloxacin. C. Telavancin + cefepime + ciprofloxacin. D. Daptomycin + cefepime + ciprofloxacin. **101. Correct answer = A.** **102.** A 23-year-old male presents with acute appendicitis that ruptures shortly after admission. He is taken to the operating room for surgery, and postsurgical cultures reveal Escherichia coli and Bacteroides fragilis, susceptibilities pending. Which of the following provides adequate empiric coverage of these two pathogens? A. Cefepime. B. Piperacillin/tazobactam. C. Aztreonam. D. Ceftaroline. **102. Correct answer = B.** **103.** A 68-year-old male presents from a nursing home with fever, increased urinary frequency and urgency, and mental status changes. He has a penicillin allergy of anaphylaxis. Which of the following β-lactams is the most appropriate choice for gram-negative coverage of this patient's urinary tract infection? A. Cefepime. B. Ertapenem. C. Aztreonam. D. Ceftaroline. **103. Correct answer = C.** **104.** A 25-year-old male presents to the urgent care center with a painless sore on his genitals that started 1 to 2 weeks ago. He reports unprotected sex with a new partner about a month ago. A blood test confirms the patient has Treponema pallidum. Which of the following is the drug of choice for the treatment of this patient's infection as a single dose? A. Benzathine penicillin G. B. Ceftriaxone. C. Aztreonam. D. Vancomycin. **104. Correct answer = A.** **105.** A 20-year-old female presents to the emergency room with headache, stiff neck, and fever for 2 days and is diagnosed with meningitis. Which of the following agents is the best choice for the treatment of meningitis in this patient? A. Cefazolin. B. Cefdinir. C. Cefotaxime. D. Cefuroxime axetil. **105. Correct answer = C.** **106.** Which of the following cephalosporins has **activity** **against gram-negative anaerobic pathogens like Bacteroides fragilis?** A. Cefoxitin. B. Cefepime. C. Ceftriaxone. D. Cefazolin. **106. Correct answer = A.** **107**. In which of the following cases would it be appropriate to use telavancin? A. A 29-year-old pregnant female with ventilator-associated pneumonia. B. A 76-year-old male with hospital-acquired pneumonia also receiving amiodarone for atrial fibrillation. C. A 36-year-old male with cellulitis and abscess growing MRSA. D. A 72-year-old female with a diabetic foot infection growing MRSA who has moderate renal dysfunction. **107. Correct answer = C.** **108.** An 18-year-old female presents to the urgent care clinic with urinary frequency, urgency, and fever for the past 3 days. Based on symptoms and a urinalysis, she is diagnosed with a urinary tract infection. Cultures reveal **[Enterococcus faecalis that is pan sensitiv]**e. Which of the following is an appropriate oral option to treat the urinary tract infection in this patient? A. Cephalexin. B. Vancomycin. C. Cefdinir. D. Amoxicillin. **108. Correct answer = D.** **109**. A 24-year-old pregnant female presents to the urgent care clinic with fever, frequency, and urgency. She is diagnosed with a urinary tract infection (UTI). Based on potential harm to the fetus, which of the following medications should be avoided in treating her UTI? A. Nitrofurantoin. B. Amoxicillin. C. Cephalexin. D. Tobramycin. **109. Correct answer = D.** **110.** Which of the following is the primary method of β-lactam resistance with Streptococcus pneumoniae? A. Modification of target site. B. Decreased drug levels due to changes in permeability. C. Decreased drug levels due to an efflux pump. D. Enzymatic inactivation. **110. Correct answer = A.**. **111.** Which of the following agents is considered a narrowspectrum antibiotic? A. Ceftriaxone. B. Ciprofloxacin. C. Isoniazid. D. Imipenem. **111. Correct answer = C.** **112.** Which of the following antibiotics exhibits **concentration dependent killing**? A. Clindamycin. B. Linezolid. C. Vancomycin. D. **Daptomycin**. **112. Correct answer = D.** **113.** Which of the following antibiotics exhibits a long **post antibiotic effect that permits** once-daily dosing? A. Gentamicin. B. Penicillin G. C. Vancomycin. D. Aztreonam. **113. Correct answer = A.** **114.** A 58-year-old male with a history of **hepatitis C, cirrhosis, and ascites** presents with spontaneous bacterial peritonitis. Which of the following antibiotics requires close monitoring and dosing adjustment in this patient given his liver disease? A. Penicillin G. B. Tobramycin. C. Erythromycin. D. Vancomycin. **114. Correct answer = C.** **115**. Which of the following antibiotics is considered safe to use in neonates? A. Chloramphenicol. B. Sulfamethoxazole/trimethoprim. C. Tetracycline. D. Penicillin G. **115. Correct answer = D.** **116.** All of the following factors influence the penetration and concentration of an antibacterial agent in the cerebrospinal fluid except: A. Lipid solubility of the drug. B. Minimum inhibitory concentration of the drug. C. Protein binding of the drug. D. Molecular weight of the drug. **116. Correct answer = B.** **117**. A 72-year-old male presents with fever, cough, malaise, and shortness of breath. His chest x-ray shows bilateral infiltrates consistent with pneumonia. Bronchial wash cultures reveal Pseudomonas aeruginosa sensitive to cefepime. **Which of the following is the best dosing scheme for cefepime based** on the drug's timedependent bactericidal activity? A. 1 g every 6 hours given over 30 minutes. B. 2 g every 12 hours given over 3 hours. C. 4 g every 24 hours given over 30 minutes. **D. 4 g given as continuous infusion over 24 hours.** **117. Correct answer = D.** **118**. Which of the following adverse drug reactions precludes a patient from being **challenged with that drug in the future**? A. Itching/rash from penicillin. B. Stevens-Johnson syndrome from sulfamethoxazole--trimethoprim. C. Gastrointestinal (GI) upset from clarithromycin. D. Clostridium difficile superinfection from moxifloxacin. **118. Correct answer = B.** **119.** A 2-year-old child is brought to the hospital after ingesting pills that a parent had used for bacterial dysentery **when traveling outside the United States**. The child has been vomiting for more than 24 h and has had diarrhea with green stools. He is now lethargic with an ashen color. Other signs and symptoms include hypothermia, hypotension, and abdominal distention. The drug most likely to be the cause of this problem is \(A) Ampicillin \(B) Chloramphenicol \(C) Clindamycin \(D) Doxycycline \(E) Erythromycin **119. The answer is B.** **120**. The mechanism of antibacterial **action of doxycycline involves** \(A) Antagonism of bacterial translocase activity \(B) Binding to a component of the 50S ribosomal subunit \(C) Inhibition of DNA-dependent RNA polymerase \(D) Interference with binding of aminoacyl-tRNA to bacterial ribosomes \(E) Selective inhibition of ribosomal peptidyl transferases **120. The answer is D.** **121**. Clarithromycin and erythromycin have very similar spectra of antimicrobial activity. The major advantage of clarithromycin is that it \(A) Does not inhibit hepatic drug-metabolizing enzymes \(B) Eradicates mycoplasmal infections in a single dose \(C) Has greater activity against M avium-intracellulare complex \(D) Is active against methicillin-resistant strains of staphylococci \(E) Is active against strains of streptococci that are resistant to erythromycin **121. The answer is C.** **122.** The primary mechanism of resistance of gram-positive **organisms to erythromycin is** \(A) Decreased activity of uptake mechanisms \(B) Methylation of binding sites on the 50S ribosomal subunit \(C) Formation of drug-inactivating acetyltransferases \(D) Formation of esterases that hydrolyze the lactone ring \(E) Decreased drug permeability of the cytoplasmic membrane **122. The answer is B.** **123.** A 26-year-old woman was treated for a suspected chlamydial infection at a neighborhood clinic. She was given a prescription for oral doxycycline to be taken for 10 d. Three weeks later, she returned to the clinic with a mucopurulent cervicitis. On questioning she admitted not having the prescription filled. The best course of action at this point would be to \(A) Delay drug treatment until the infecting organism is identified \(B) Rewrite the original prescription for oral doxycycline \(C) Treat her in the clinic with a single oral dose of cefixime \(D) Treat her in the clinic with a single oral dose of azithromycin \(E) Write a prescription for oral erythromycin for 10 d **123. The answer is D.** **124.** A 55-year-old patient with a prosthetic heart valve is to undergo a periodontal procedure involving scaling and root planing. Several years ago, the patient had a severe allergic reaction to procaine penicillin G. Regarding prophylaxis against bacterial endocarditis, which one of the following drugs taken orally is most appropriate? \(A) Amoxicillin 10 min before the procedure \(B) Clindamycin 1 h before the procedure \(C) Erythromycin 1 h before the procedure and 4 h after the procedure \(D) Vancomycin 15 min before the procedure \(E) No prophylaxis is needed because this patient is in the negligible risk category **124. The answer is B.** **125.** A 24-year-old woman comes to a clinic with complaints of dry cough, headache, fever, and malaise, which have lasted 3 or 4 d. She appears to have some respiratory difficulty, and chest examination reveals rales but no other obvious signs of pulmonary involvement. However, extensive patchy infiltrates are seen on chest x-ray film. Gram stain of expectorated sputum fails to reveal any bacterial pathogens. The patient mentions that a colleague at work has similar symptoms to those she is experiencing. The patient has no history of serious medical problems. She **takes loratadine for allergies and supplementary iron tablets, and she drinks at least 6 cups of caffeinated coffee per day**. The physician makes an initial diagnosis of community-acquired pneumonia. Regarding the treatment of this patient, which of the following drugs is most suitable? \(A) Amoxicillin \(B) Clindamycin \(C) Doxycycline \(D) Linezolid \(E) Vancomycin **125. The answer is C.** **126.** If the patient is treated with the **macrolide erythromycin, he should**: \(A) Avoid exposure to sunlight \(B) Avoid taking supplementary iron tablets \(C) Decrease her intake of caffeinated beverages \(D) Discontinue loratadine temporarily \(E) Have her plasma urea nitrogen or creatinine checked before treatment **126. The answer is C.** **127.** A 5 day course **of treatment for community-acquired** pneumonia would be effective with little risk of drug interactions if the drug prescribed is \(A) Ampicillin **(B) Azithromycin** \(C) Clindamycin \(D) Erythromycin \(E) Vancomycin **127. The answer is B.** **128.** Concerning quinupristin-dalfopristin, which statement is accurate? \(A) Active in treatment of infections caused by E. faecalis \(B) Bacteriostatic \(C) Hepatotoxicity has led to FDA drug alerts \(D) Induce formation of hepatic drug-metabolizing enzymes \(E) Used in management of **128. The answer is E.** **129.** Regarding the mechanism of action of aminoglycosides, the drugs \(A) Are bacteriostatic \(B) Bind to the 50S ribosomal subunit \(C) Cause misreading of the code on the mRNA template \(D) Inhibit peptidyl transferase \(E) Stabilize polysomes **129. The answer is C.** **130.** A 50-kg patient with creatinine clearance of 80 mL/min has a gram-negative infection. Amikacin is administered intramuscularly at dose of 5 mg/kg every 8 h, and the patient begins to respond. After 2 d, creatinine clearance declines to 40 mL/min. Assuming that no information is available about amikacin plasma levels, what would be the most reasonable approach to management of the patient at this point? \(A) Administer 5 mg/kg every 12 h \(B) Decrease the dosage to a daily total of 200 mg \(C) Decrease the dosage to 125 mg every 8 h \(D) Discontinue amikacin and switch to gentamicin \(E) Maintain the patient on the present dosage and test auditory function **130. The answer is C.** **131.** All of the following statements about the clinical uses of **the aminoglycosides are accurate EXCEP**T \(A) Effective in the treatment of infections caused by Bacteroides fragilis \(B) Gentamicin is used with ampicillin for synergistic effects in the treatment of enterococcal endocarditis \(C) Netilmicin is more likely to be effective than streptomycin in the treatment of a hospital-acquired infection caused by Serratia marcescens \(D) Often used in combination with cephalosporins in the empiric treatment of life-threatening bacterial infections \(E) Owing to their polar nature, aminoglycosides are not absorbed after oral administration **131. The answer is A.** **132.** Which statement is accurate regarding the **antibacterial action of gentamicin**? \(A) Antibacterial activity is often reduced by the presence of an inhibitor of cell wall synthesis \(B) Antibacterial action is not concentration-dependent (C) Antibacterial action is time-dependent \(C) Efficacy is directly proportional to the duration of time that the plasma level is greater than the minimal inhibitory concentration \(D) Gentamicin continues to exert antibacterial effects even after plasma levels decrease below detectable levels **132. The answer is D**. **133.** An adult patient (weight 60 kg) has bacteremia suspected to be due to a gram-negative rod. Tobramycin is to be administered using a once-daily dosing regimen, and the loading dose must be calculated to achieve a peak plasma level of 20 mg/L. Assume that the patient has normal renal function. Pharmacokinetic parameters of tobramycin in this patient are as follows: Vd = 20 L; t1/2 = 3 h; CL = 80 mL/min. What loading dose should be given? \(A) 100 mg \(B) 200 mg \(C) 400 mg \(D) 600 mg \(E) 800 mg **133. The answer is C.** **134.** A 67-year-old man is seen in a hospital emergency department complaining of pain in and behind the right ear. Physical examination shows edema of the external otic canal with purulent exudate and weakness of the muscles on the right side of the face. The patient informs the physician that he is a diabetic. Gram stain of the exudate from the ear shows many **polymorphonucleocytes and gram-negative** rods, and samples are sent to the microbiology laboratory for culture and drug susceptibility testing. A preliminary diagnosis is made of external otitis. **At this point, which of the following is most appropriate?** **(A)** Amikacin should be administered by intramuscular injection, and the patient should be sent home **(B)** Analgesics should be prescribed for pain, but antibiotics should be withheld pending the results of cultures **(C)** Oral cefaclor should be prescribed together with analgesics, and the patient should be sent home **(D) The patient should be hospitalized and treatment started with gentamicin plus ticarcillin** **(E)** The patient should be hospitalized and treatment started with intravenous imipenem-cilastatin **134. The answer is D.** **135.** Regarding the toxicity of aminoglycosides, which statement is accurate? **(A)** Gentamicin and tobramycin are the least likely to cause renal damage **(B)** Ototoxicity due to amikacin and gentamicin includes vestibular dysfunction that is often irreversible **(C)** Ototoxicity is reduced if loop diuretics are used to facilitate aminoglycoside renal excretion **(D)** Skin reactions are rare with use of topical neomycin **(E)** With traditional dosage regimens, the earliest sign of nephrotoxicity is a reduced blood creatinine **135. The answer is B.** **136.** This drug has characteristics almost identical to those of gentamicin but has **much weaker activity** in combination with penicillin against enterococci. **(A)** Amikacin **(B)** Erythromycin **(C)** Netilmicin **(D)** Spectinomycin **(E)** Tobramycin **136. The answer is E.** **137.** Your 23-year-old female patient is pregnant and has gonorrhea. The medical history includes anaphylaxis following exposure to amoxicillin. The most appropriate drug to use is **(A)** Azithromycin **(B)** Cefixime **(C)** Ceftriaxone **(D)** Ciprofloxacin **(E)** Doxycycline **137. The answer is A.** **138.** Which statement about "once-daily" dosing with aminoglycosides is not accurate? **(A)** Convenient for outpatient therapy **(B)** Dosage adjustment is less important in renal insufficiency **(C)** Less nursing time is required for drug administration **(D)** Often less toxic than conventional (multiple) dosing regimens **(E)** Underdosing is less of a problem **138. The answer is B.** **139.** A 24-year-old woman has returned from a vacation abroad suffering from traveler's diarrhea, and her problem **has not responded to antidiarrheal drugs**. A pathogenic gram-negative bacillus is suspected. Which drug is most likely to be effective in the treatment of this patient? **(A)** Amoxicillin **(B)** Ciprofloxacin **(C)** Sulfacetamide **(D)** Trimethoprim **(E)** Vancomycin **139. The answer is B.** **140.** Which statement about the clinical use of **sulfonamides is false**? **(A)** Active against *C. trachomatis* and can be used topically for the treatment of *chlamydial* infections of the eye **(B)** Are not effective as sole agents in the treatment of prostatitis **(C)** Effective in *Rocky Mountain spotted* fever in patients allergic to tetracyclines **(D)** Resistance can occur in some strains of bacteria because of increased production of PABA **(E)** Some resistant bacterial strains exhibit decreased intracellular accumulation of sulfonamides **140. The answer is C.** **\ **1. Trimethoprim-sulfamethoxazole is established to be effective against which of the following opportunistic infections in the AIDS patient? \(A) Cryptococcal meningitis \(B) Disseminated herpes simplex \(C) Oral candidiasis \(D) Toxoplasmosis \(E) Tuberculosis **The answer is D** 2\. A 24-year-old woman has returned from a vacation abroad suffering from traveler's diarrhea, and her problem has not responded to antidiarrheal drugs. A pathogenic gram-negative bacillus is suspected. Which drug is most likely to be effective in the treatment of this patient? \(A) Amoxicillin \(B) Ciprofloxacin \(C) Sulfacetamide \(D) Trimethoprim \(E) Vancomycin **The answer is B** 3\. Which statement about the clinical use of sulfonamides is false? \(A) Active against C. trachomatis and can be used topically for the treatment of chlamydial infections of the eye \(B) Are not effective as sole agents in the treatment of prostatitis \(C) Effective in Rocky Mountain spotted fever in patients allergic to tetracyclines \(D) Resistance can occur in some strains of bacteria because of increased production of PABA \(E) Some resistant bacterial strains exhibit decreased intracellular accumulation of sulfonamides **The answer is C** 4\. A 31-year-old man has gonorrhea. He has no drug allergies, but a few years ago acute hemolysis followed use of an antimalarial drug. The physician is concerned that the patient has an accompanying urethritis caused by C. trachomatis, although no cultures or enzyme tests have been performed. Which of the following drugs will be reliably effective against both gonococci and C trachomatis and safe to use in this patient? \(A) Cefixime \(B) Ciprofloxacin \(C) Spectinomycin \(D) Sulfamethoxazole-trimethoprim \(E) None of the above **The answer is E** 5\. Which statement about the fluoroquinolones is accurate? \(A) A fluoroquinolone is the drug of choice for treatment of an uncomplicated urinary tract infection in a 7-year-old girl \(B) Antacids increase the oral bioavailability of fluoroquinolones \(C) Gonococcal resistance to fluoroquinolones may involve changes in DNA gyrase \(D) Modification of moxifloxacin dosage is required in patients when creatinine clearance is less than 50 mL/min \(E) The fluoroquinolones are contraindicated in patients with hepatic dysfunction **The answer is C** 6\. A 55-year-old man complains of periodic bouts of diarrhea with lower abdominal cramping and intermittent rectal bleeding. Seen in the clinic, he appears well nourished, with blood pressure in the normal range. Examination reveals moderate abdominal pain and tenderness. His current medications are limited to loperamide for his diarrhea. Sigmoidoscopy reveals mucosal edema, friability, and some pus. Laboratory findings include mild anemia and decreased serum albumin. Microbiologic examination via stool cultures and mucosal biopsies do not reveal any evidence for bacterial, amebic, or cytomegalovirus involvement. The most appropriate drug to use in this patient is \(A) Amoxicillin \(B) Ciprofloxacin \(C) Doxycycline \(D) Sulfasalazine \(E) Trimethoprim-sulfamethoxazole **The answer is D** 7\. Which adverse effect is most likely to occur with sulfonamides? \(A) Fanconi's aminoaciduria syndrome \(B) Hematuria \(C) Kernicterus in the newborn \(D) Neurologic dysfunction \(E) Skin reactions **The answer is E** 8\. Which drug is effective in the treatment of nocardiosis and, in combination with pyrimethamine, is prophylactic against Pneumocystis jiroveci infections in AIDS patients? \(A) Amoxicillin \(B) Ciprofloxacin \(C) Clindamycin \(D) Sulfadiazine \(E) Trimethoprim **The answer is D** 9\. Which statement about ciprofloxacin is accurate? \(A) Active against most MRSA strains of staphylococci \(B) Antagonism occurs if it is used with inhibitors of dihydrofolate reductase \(C) During treatment, tendinitis and even tendon rupture may occur \(D) Most "first-time" urinary tract infections are resistant to ciprofloxacin \(E) Organisms associated with middle ear infections are highly resistant **The answer is C** 10\. Supplementary folinic acid may prevent anemia in folate deficient persons who use this drug; it is a weak base achieving tissue levels similar to those in plasma \(A) Ciprofloxacin \(B) Moxifloxacin \(C) Sulfacetamide \(D) Sulfamethoxazole \(E) Trimethoprim **The answer is E** **True/false questions** **CHF** **1.** Vasodilator therapy with vasodilators nitroprusside or nitroglycerin is often used for acute severe failure with congestion and can be dramatically effective, especially in cases in which increased afterload is a major factor. **A.**True **B.** False **2.** Correction of potassium deficiency (caused, eg, by diuretic use) is of no value in chronic digitalis intoxication. **A.**True **B.** False **3.** Considerable evidence indicates that angiotensin antagonists, certain β-adrenoceptor blockers, and the aldosterone antagonists spironolactone and eplerenone also have long-term beneficial effects in congestive heart failure. **A.**True **B.** False **4.** Chronic heart failure is best treated with loop diuretic; if severe, a prompt-acting positive inotropic agent such as a β agonist or phosphodiesterase inhibitor and and vasodilators. **A.**True **B.** False **5.** Beta blockers are of no value in acute failure and may be detrimental if systolic dysfunction is marked. **A.**True **B.** False **IHD** *β-Blockers* **1.** Conditions that discourage the use of β-blockers include reversible bronchospastic disease such as asthma, second- and third-degree heart block, and severe peripheral vascular disease. **A. True** **B. False** **2.** **Nebivolol is a selective blocker of β~1~ receptors,** which also increases the production of nitric oxide, leading to vasodilation. **A. True** **B. False** **3.** The β-blockers reduce blood pressure primarily by decreasing cardiac output. They may also decrease sympathetic **outflow from the central nervous system (CNS) and inhibit the** release of renin from the kidneys, thus decreasing the formation of angiotensin II and the secretion of aldosterone. **A. True** **B. False** **4.** Beta blockers are used in angina only for acute attack; they are of no value in an prophylactic therapy. **A.**True **B.** **False** **5.** Beta blockers initially decrease cardiac output, but in chronic use their action may include an increase in **vascular resistance as a contributing effect**. **A.True** **B. False** *Calcium channel-blockers* **6.** *Verapamil* is the most selective of any calcium channel blocker and has significant **effects only on cardiac m**uscle cells. **A. True** **B. False** **7.** In the treatment of effort-induced angina, calcium channel blockers reduce myocardial oxygen consumption by increasing vascular resistance, thereby increasing afterload. **A. True** **B. False** **8.** *Diltiazem* affects both cardiac and vascular smooth muscle cells, but it has a less pronounced negative inotropic effect on the heart compared to that of ***verapamil. Diltiazem* has a favorable side effect profile.** **A. True** **B. False** **9.** Calcium blockers relax blood vessels and, to a lesser extent, the uterus, bronchi, and gut. **A.True** **B.** False **10.** Nifedipine and other dihydropyridines evoke greater vasodilation then verapamil and diltiazem , and the resulting sympathetic reflex prevents bradycardia and may actually increase heart rate. **A.True** **B.** False **Antihypertensives** **1.** The ACE inhibitors are useful in heart failure and diabetes as well as in hypertension. **A.**True **B.** False **2.** Alpha1-selective agents (eg, prazosin, doxazosin, terazosin) are the most effective antihypertensive drugs. **A.**True **B.** False **3.** Compensatory responses to diuretics, beta blockers and angiotensin-**renin antagonists is minimal.** **A.**True **B.** False **4.** Less than 20% of cases of hypertension are due to ("secondary" to) factors that can be clearly defined and corrected. This type of hypertension is associated with pheochromocytoma, coarctation of the aorta, renal vascular disease, adrenal cortical tumors, and a few other rare conditions. **A.**True **B.** False **5.** **Nitroprusside is oral**, short-acting vasodilator used in hypertensive emergencies. **A.**True **B.** False **Diuretics** **1.** Thiazides are not useful in combination therapy with a other antihypertensive agents, including β-blockers, ACE inhibitors, ARBs, and potassium-sparing diuretics. **A. True** **B. False** **2.** With the exception of *metolazone*, thiazide diuretics are not effective in patients with inadequate kidney function (estimated glomerular filtration rate less than 30 mL/min/m2). **A. True** **B. False** **3.** Loop diuretics increase renal vascular resistance and decrease renal blood flow. **A. True** **B. False** **4.** The loop diuretics (*furosemide*, *torsemide*, *bumetanide*, and *ethacrynic acid*) act promptly by blocking sodium and chloride reabsorption in the kidneys, even in patients with poor renal function or those who have not responded to thiazide diuretics. **A. True** **B. False** **5.** Thiazide diuretics can induce hypokalemia, hyperuricemia and, to a lesser extent, hyperglycemia in some patients. **A. True** **B. False** **6.** Like thiazides, loop diuretics can cause hypokalemia. However, unlike thiazides, loop diuretics increase the Ca^2+^ content of urine, whereas thiazide diuretics decrease it. **A. True** **B. False** **7.** Potassium-sparing diuretics are never used in combination with loop diuretics and thiazides **A. True** **B. False** **8.** Aldosterone antagonists doesn't have the additional benefit of diminishing the cardiac remodeling that occurs in heart failure. **A. True** **B. False** **9.** *Amiloride* and *triamterene* (inhibitors of epithelial sodium transport at the late distal and collecting ducts) as well as *spironolactone* and *eplerenone* (aldosterone receptor antagonists) increase potassium loss in the urine. **A. True** **B. False** **Coagulation** **1.** Heparin is highly basic and can be neutralized by acidic molecules as protamine. **A.**True **B.** False **2.** Prolonged use of unfractionated **heparin is associated with osteoporosis**. **A.**True **B.** False **3.** Unfractionated heparin binds to endogenous antithrombin III (ATIII) via a key **pentasaccharide sequence.** **A.**True **B.** False **4.** Anticlotting drugs are divided in three groups : antiplatelets, anticoagulants and thrombolitics. **A.**True **B.** False **5.** Enoxaparin and other LMW heparins have less bioavailability and **shorter durations of action than unfractionated heparin** **A.**True **B.** False **Antimicrobials** **1.** Acutely ill patients with infections of unknown origin---for example, a neutropenic patient or a patient with meningitis - require immediate treatment. **A. True** **B. False** **2.** Bacteriostatic drugs arrest the growth and replication of bacteria at serum (or urine) levels achievable in the patient, thus limiting the spread of infection until the immune system attacks, immobilizes, and eliminates the pathogen. **A. True** **B. False** **3**. Bacteric